Какой подход выбрать с вертикальной пружиной?

Допустим, у нас есть пружина, висящая вертикально с жесткостью пружины. к прикрепленный к блоку массы м . Система находится в покое.

Затем вы тянете массу вниз, растягивая пружину на расстояние Икс , то отпустите. Пружина, конечно, вернется в исходное положение. Какова скорость тела в начальной точке покоя?

Чтобы решить эту проблему, я использовал два подхода, но я не уверен, какой из них правильный. Во-первых, это рабочий подход. Когда блок возвращается на свое старое место, он остается прежним, за исключением того, что теперь он имеет новую энергию, полученную от предыдущего расширения, так что я могу сказать...

Вт "=" Ф д или Вт "=" 1 2 * к * Икс 2 .

Так, 1 2 * к * Икс 2 "=" 1 2 * м * в 2 поэтому в "=" к м * а б с ( Икс ) .

Однако я не учитываю гравитационный потенциал, который меня беспокоит.

Если мы это сделаем, то можем сказать, что в остальных местах энергия просто м г час где час является Икс .

Внизу энергия просто 1 2 * к * Икс 2 так...

м г Икс + 1 2 м * в 2 "=" 1 2 к * Икс 2 так в "=" к Икс 2 2 г м Икс м

Какой подход выбрать?

Вы не держите свои точки отсчета последовательными. Если вы принимаете Икс "=" 0 как высота в состоянии покоя, то потенциальная энергия в состоянии покоя равна м г 0 "=" 0 , а потенциальная энергия при опущенной пружине представляет собой сумму энергии пружины и энергии, отдаваемой массой: Вт "=" 1 2 к Икс 2 м г | Икс | .
@TimWescott Значит ли это, что второй подход правильный? Я могу просто переместить м г Икс на другую сторону и найти v, что дает то же выражение?
Покрутите, посмотрите, как это работает.
Я должен упомянуть, что ваш второй подход несет в себе все признаки, которые для меня будут включать в себя несколько попыток, обнаружив, что я оставил «i без точек», «t не зачеркнут, или я неправильно подсчитал изменения знака и имею «+» вместо «-» должен быть и наоборот. Так что крутите, осторожно .
@TimWescott Вы говорите так, как будто нет способа проверить фактическое решение.

Ответы (1)

Оба подхода на самом деле одинаковы, если вы делаете их правильно.

Сначала я рассмотрю ваш второй случай. Вы правильно используете закон сохранения энергии и говорите, что потенциальная энергия, запасенная пружиной в самой нижней точке, равна сумме кинетической энергии и потенциальной энергии силы тяжести в точке равновесия. Так что вы были правы с уравнением

1 2 к у 2 "=" м г у + 1 2 м в 2

Давайте теперь посмотрим на первый случай, но давайте сделаем это правильно. Мы знаем, что чистая работа, совершаемая над массой, равна изменению ее кинетической энергии:

Вт сеть "=" Вт сила тяжести + Вт весна "=" Δ К "=" 1 2 м в 2 0

Мы можем легко определить работу силы тяжести и силы пружины, используя определение работы Вт "=" Ф д у

Вт сила тяжести "=" у 0 ( м г ) д у "=" м г у
Вт весна "=" у 0 ( к у ) д у "=" 1 2 к у 2
Собрав все вместе, мы имеем
Вт сеть "=" м г у + 1 2 к у 2 "=" 1 2 м в 2

Вы можете видеть, что это точно так же, как ваш второй случай. Так что оба метода, которые вы предложили, абсолютно одинаковы. Проблема с первым случаем в вашем вопросе такая же, как вы сказали. Вы не включили работу силы тяжести.